Amount of Terms Required to Find a Sum at an Indicated Accuracy

In summary, the conversation discusses how to show that the series \sum^{\infty}_{n=1}\frac{(-1)^{n}}{n*9^{n}} is convergent and how many terms are needed to find the sum to a certain accuracy. The alternating series test is used and it is determined that n=3 terms should be added to find the sum within a margin of error of 0.0001. There is also a discussion about taking the absolute value of the terms, which is not necessary in this case.
  • #1
NastyAccident
61
0

Homework Statement


Show that the series is convergent. How many terms of the series do we need to add in order to find the sum to the indicated accuracy?

[tex]\sum^{\infty}_{n=1}\frac{(-1)^{n}}{n*9^{n}}[/tex]
(|error| < 0.0001)

Homework Equations


Alternating Series Test
General knowledge of adding series up...

The Attempt at a Solution


Alternating Series Test
1.) Limit x->infinity
[tex]\frac{(-1)^{n}}{n*9^{n}}[/tex] = 0
2.) Decreasing eventually for n (really immediately)
[tex]\sum^{\infty}_{n=1}\frac{(-1)^{n+1}}{(n+1)*9^{n+1}} < \sum^{\infty}_{n=1}\frac{(-1)^{n}}{n*9^{n}}[/tex]

By the Alternating Series Test, this series ([tex]\sum^{\infty}_{n=1}\frac{(-1)^{n}}{n*9^{n}}[/tex]) is convergent.

Well, I originally choose n = 4 terms in order to find the indicated accuracy. However, that was wrong.

So, I'm sort of scratching my head as to what I did wrong...

My approach:
[tex]\sum^{\infty}_{n=1}\frac{(-1)^{n}}{n*9^{n}}[/tex]

I start listing out an's terms -
|an| < 0.0001
a1 = 0.11111111111
a2 = 0.006172839506
a3 = 0.000457247370
a4 = 0.000038103947
a5 = 0.000003387017
a6 = 0.000000313612
a7 = 0.000000029867

a4 < 0.0001, so n should equal four terms... However, it doesn't for some odd reason?

Any help/suggestions as to where I went wrong are appreciated and will be thanked!
NastyAccident
 
Last edited:
Physics news on Phys.org
  • #2
You have an alternating series, so every other an (starting from aa) should be negative. You show all of them as positive in your list.
 
  • #3
Mark44 said:
You have an alternating series, so every other an (starting from aa) should be negative. You show all of them as positive in your list.

So, in essence, I shouldn't of taken the absolute value of each of the an's and should of left it with the +/- signs?

If that is the case, then the problem should be okay for n = 3?

Since:
[tex]\sum^{\infty}_{n=1}\frac{(-1)^{n}}{n*9^{n}}[/tex]
is within
[tex]\sum^{\infty}_{n=1}\frac{(1)^{n+1}}{(n+1)*9^{(n+1)}}[/tex]
which gives me:

an < 0.0001
a1 = 0.061728395
a2 = 0.0004572473708
a3 = 0.000038103947
a4 = 0.000003387017
a5 = 0.000000313612
a6 = 0.000000029867

So, n = 3 would be correct...
 
Last edited:
  • #4
Right. You don't want the abs. value of your terms. Just add them up. Your estimate with a1 + a2 + a3 should be within .0001 of the actual value of the infinite sum. You did add up the three terms, right?
 

1. What is the meaning of "Amount of Terms Required to Find a Sum at an Indicated Accuracy"?

The amount of terms required to find a sum at an indicated accuracy refers to the number of terms needed in a series or sequence to accurately estimate the sum of that series or sequence with a given degree of precision.

2. Why is it important to know the amount of terms required to find a sum at an indicated accuracy?

Knowing the amount of terms required to find a sum at an indicated accuracy allows us to determine the level of precision needed in our calculations and helps us understand the convergence of a series or sequence. It also allows us to determine the computational time and resources needed for accurate calculations.

3. How is the amount of terms required to find a sum at an indicated accuracy calculated?

The amount of terms required to find a sum at an indicated accuracy is calculated using mathematical formulas or algorithms specific to the type of series or sequence being evaluated. These formulas take into account the desired level of accuracy and the properties of the series or sequence.

4. Can the amount of terms required to find a sum at an indicated accuracy vary for different series or sequences?

Yes, the amount of terms required to find a sum at an indicated accuracy can vary depending on the properties of the series or sequence being evaluated. Some series or sequences may converge quickly, requiring fewer terms for a given accuracy, while others may converge slowly, requiring more terms.

5. How does the level of accuracy affect the amount of terms required to find a sum?

The higher the desired level of accuracy, the more terms will be required to find a sum at an indicated accuracy. This is because as the level of precision increases, the margin of error decreases, and more terms are needed to accurately estimate the sum of a series or sequence.

Similar threads

  • Calculus and Beyond Homework Help
Replies
1
Views
252
  • Calculus and Beyond Homework Help
Replies
14
Views
1K
  • Calculus and Beyond Homework Help
Replies
7
Views
1K
  • Calculus and Beyond Homework Help
Replies
12
Views
982
  • Calculus and Beyond Homework Help
Replies
4
Views
2K
  • Calculus and Beyond Homework Help
Replies
7
Views
1K
  • Calculus and Beyond Homework Help
Replies
13
Views
1K
  • Calculus and Beyond Homework Help
Replies
15
Views
1K
  • Calculus and Beyond Homework Help
Replies
9
Views
2K
  • Calculus and Beyond Homework Help
Replies
6
Views
1K
Back
Top